{ "cells": [ { "cell_type": "markdown", "id": "3960a901", "metadata": {}, "source": [ "# Centre Universitaire de Mila" ] }, { "cell_type": "markdown", "id": "c4568d4a", "metadata": {}, "source": [ "## Master 1 (STIC & I2A), Matière: Apprentissage Automatique" ] }, { "cell_type": "markdown", "id": "a8bedc4a", "metadata": {}, "source": [ "## Travaux pratiques N°7: Réseaux de neurones" ] }, { "cell_type": "markdown", "id": "f57e8c63", "metadata": {}, "source": [ "## Enoncé" ] }, { "cell_type": "markdown", "id": "88db096e", "metadata": {}, "source": [ "# 1. Introduction" ] }, { "cell_type": "markdown", "id": "19e3b7ce", "metadata": {}, "source": [ "Un MLP (Multilayer Perceptron) est un algorithme d'apprentissage automatique supervisé (ML) qui appartient à la classe des réseaux de neurones artificiels à propagation avant (feedforward artificial neural networks). L'algorithme est essentiellement entraîné sur les données afin d'apprendre une fonction. Étant donné un ensemble de caractéristiques et une variable cible (par exemple, des étiquettes), il apprend une fonction non linéaire pour la classification ou la régression. Dans ce TP, nous nous concentrerons uniquement sur le cas de la classification." ] }, { "cell_type": "markdown", "id": "509d5900", "metadata": {}, "source": [ "# 2.\tL’ensemble de données (dataset)" ] }, { "cell_type": "markdown", "id": "0c34d08f", "metadata": {}, "source": [ "Pour cet exemple pratique, nous utiliserons l'ensemble de données MNIST. La base de données MNIST est une célèbre base de données de chiffres manuscrits utilisée pour entraîner plusieurs modèles en apprentissage automatique. Il y a des images manuscrites de 10 chiffres différents, donc le nombre de classes est de 10 (voir Figure 1)." ] }, { "attachments": { "mnist_dataset.png": { "image/png": "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" } }, "cell_type": "markdown", "id": "32be3cca", "metadata": {}, "source": [ "![mnist_dataset.png](attachment:mnist_dataset.png)" ] }, { "cell_type": "markdown", "id": "77ea4273", "metadata": {}, "source": [ "### Notes :\n", "* Comme nous travaillons avec des images, celles-ci sont représentées par des matrices 2D et la dimension initiale des données est de 28 par 28 pour chaque image (28x28 pixels). Les images 2D sont ensuite aplaties et donc représentées par des vecteurs à la fin. Chaque image 2D est transformée en un vecteur 1D de dimension [1, 28x28] = [1, 784]. Finalement, notre ensemble de données comporte 784 caractéristiques/variables/colonnes.\n", "\n", "* Lien pour télécharger les données:\n", "https://datahub.io/machine-learning/mnist_784/r/mnist_784.csv" ] }, { "cell_type": "markdown", "id": "015d8685", "metadata": {}, "source": [ "# 3. Importation et préparation des données" ] }, { "cell_type": "markdown", "id": "1359427a", "metadata": {}, "source": [ "### Question:\n", "Ecrire les commandes Python qui permettent de :\n", "* Charger les données à partir du dataset mnist\n", "* Séparer les données d'entrées et cibles\n", "* Normaliser l'intensité des images pour la ramener dans l'intervalle [0,1]. " ] }, { "cell_type": "code", "execution_count": 1, "id": "dead834d", "metadata": {}, "outputs": [], "source": [ "import numpy as np\n", "import pandas as pd\n", "import matplotlib.pyplot as plt\n", "from sklearn.neural_network import MLPClassifier\n", "\n", "# Charger les données\n", "mnist_data = ...\n", "\n", "# Convertir à numpy\n", "mnist_data = ...\n", "\n", "# Séparer les données d'entrées et cibles\n", "X = ...\n", "y = ...\n", "\n", "# Normaliser l'intensité des images pour la ramener dans l'intervalle [0,1] car 255 est le max (blanc).\n", "X = ..." ] }, { "cell_type": "markdown", "id": "72d0ac92", "metadata": {}, "source": [ "Rappelez-vous que chaque image 2D est maintenant transformée en un vecteur 1D de dimension [1, 28x28] = [1, 784]. \n", "\n", "### Question:\n", "Ecrire la commande Python qui permet de vérifier les dimensions ainsi que les intervalles des données." ] }, { "cell_type": "code", "execution_count": 2, "id": "9a7b3b5d", "metadata": {}, "outputs": [ { "name": "stdout", "output_type": "stream", "text": [ "(70000, 785)\n", "(70000, 784)\n", "(70000,)\n", "Intervalle X : [0.0,1.0]\n", "Intervalle y : [0,9]\n" ] } ], "source": [ "print(...)\n", "print(...)\n", "print(...)\n", "print('Intervalle X : ...')\n", "print('Intervalle y : ...')" ] }, { "cell_type": "markdown", "id": "c22faba5", "metadata": {}, "source": [ "Cela renvoie : (70000, 784). Nous avons 70 000 images aplaties (échantillons), chacune contenant 784 pixels (28*28=784) (variables/caractéristiques). Ainsi, la matrice de poids de la couche d'entrée aura une forme de 784 x #neurons_in_1st_hidden_layer. La matrice de poids de la couche de sortie aura une forme de #neurons_in_3rd_hidden_layer x #number_of_classes." ] }, { "cell_type": "markdown", "id": "a6591938", "metadata": {}, "source": [ "# 4.\tEntraînement du modèle" ] }, { "cell_type": "markdown", "id": "49647221", "metadata": {}, "source": [ "### Question:\n", "Maintenant, construire le modèle de réseaux de neurones, entraîner-le et effectuer la classification. Utiliser 3 couches cachées avec respectivement 50, 20 et 10 neurones. De plus, nous fixerer le nombre maximal d'itérations à 100 et le taux d'apprentissage à 0,1.\n", "\n", "Rappelez-vous que chaque image 2D est maintenant transformée en un vecteur 1D de dimension [1, 28x28] = [1, 784]. " ] }, { "cell_type": "code", "execution_count": 3, "id": "47f7c7bc", "metadata": {}, "outputs": [ { "name": "stdout", "output_type": "stream", "text": [ "Iteration 1, loss = 0.42635367\n", "Iteration 2, loss = 0.15133481\n", "Iteration 3, loss = 0.11926082\n", "Iteration 4, loss = 0.10128421\n", "Iteration 5, loss = 0.08698448\n", "Iteration 6, loss = 0.08018627\n", "Iteration 7, loss = 0.07544472\n", "Iteration 8, loss = 0.06650726\n", "Iteration 9, loss = 0.06502276\n", "Iteration 10, loss = 0.05670472\n", "Iteration 11, loss = 0.05228727\n", "Iteration 12, loss = 0.05194876\n", "Iteration 13, loss = 0.04580530\n", "Iteration 14, loss = 0.04507070\n", "Iteration 15, loss = 0.04141424\n", "Iteration 16, loss = 0.03988480\n", "Iteration 17, loss = 0.03980626\n", "Iteration 18, loss = 0.03593785\n", "Iteration 19, loss = 0.03619045\n", "Iteration 20, loss = 0.03170852\n", "Iteration 21, loss = 0.03625169\n", "Iteration 22, loss = 0.03089522\n", "Iteration 23, loss = 0.02847495\n", "Iteration 24, loss = 0.02632563\n", "Iteration 25, loss = 0.03070260\n", "Iteration 26, loss = 0.02767076\n", "Iteration 27, loss = 0.02586306\n", "Iteration 28, loss = 0.02568441\n", "Iteration 29, loss = 0.02746646\n", "Iteration 30, loss = 0.02268341\n", "Iteration 31, loss = 0.02424414\n", "Iteration 32, loss = 0.02574972\n", "Iteration 33, loss = 0.02438313\n", "Iteration 34, loss = 0.02130451\n", "Iteration 35, loss = 0.01997140\n", "Iteration 36, loss = 0.01839737\n", "Iteration 37, loss = 0.01913580\n", "Iteration 38, loss = 0.02194003\n", "Iteration 39, loss = 0.02379174\n", "Iteration 40, loss = 0.01655622\n", "Iteration 41, loss = 0.02142640\n", "Iteration 42, loss = 0.01563293\n", "Iteration 43, loss = 0.02199450\n", "Iteration 44, loss = 0.01805852\n", "Iteration 45, loss = 0.01779430\n", "Iteration 46, loss = 0.01951887\n", "Iteration 47, loss = 0.01881607\n", "Iteration 48, loss = 0.01823314\n", "Iteration 49, loss = 0.01553454\n", "Iteration 50, loss = 0.02193452\n", "Iteration 51, loss = 0.01634748\n", "Iteration 52, loss = 0.01594851\n", "Iteration 53, loss = 0.01459823\n", "Iteration 54, loss = 0.01829314\n", "Iteration 55, loss = 0.01729073\n", "Iteration 56, loss = 0.01735602\n", "Iteration 57, loss = 0.01854976\n", "Iteration 58, loss = 0.01481691\n", "Iteration 59, loss = 0.01272347\n", "Iteration 60, loss = 0.01380166\n", "Iteration 61, loss = 0.01499409\n", "Iteration 62, loss = 0.01505173\n", "Iteration 63, loss = 0.01246990\n", "Iteration 64, loss = 0.01538067\n", "Iteration 65, loss = 0.01659721\n", "Iteration 66, loss = 0.01336988\n", "Iteration 67, loss = 0.01481459\n", "Iteration 68, loss = 0.01311940\n", "Iteration 69, loss = 0.01340073\n", "Iteration 70, loss = 0.01132432\n", "Iteration 71, loss = 0.01860893\n", "Iteration 72, loss = 0.01676240\n", "Iteration 73, loss = 0.01588535\n", "Iteration 74, loss = 0.01018722\n", "Iteration 75, loss = 0.01607946\n", "Iteration 76, loss = 0.01014969\n", "Iteration 77, loss = 0.00795249\n", "Iteration 78, loss = 0.01302940\n", "Iteration 79, loss = 0.01303363\n", "Iteration 80, loss = 0.01116531\n", "Iteration 81, loss = 0.01214103\n", "Iteration 82, loss = 0.00929063\n", "Iteration 83, loss = 0.01510738\n", "Iteration 84, loss = 0.01478250\n", "Iteration 85, loss = 0.01749228\n", "Iteration 86, loss = 0.00974255\n", "Iteration 87, loss = 0.00777282\n", "Iteration 88, loss = 0.00916963\n", "Iteration 89, loss = 0.00951186\n", "Iteration 90, loss = 0.00680212\n", "Iteration 91, loss = 0.01713153\n", "Iteration 92, loss = 0.01354235\n", "Iteration 93, loss = 0.01625461\n", "Iteration 94, loss = 0.01259690\n", "Iteration 95, loss = 0.01078507\n", "Iteration 96, loss = 0.00647461\n", "Iteration 97, loss = 0.01073508\n", "Iteration 98, loss = 0.01069441\n", "Iteration 99, loss = 0.00916135\n", "Iteration 100, loss = 0.01224429\n" ] }, { "name": "stderr", "output_type": "stream", "text": [ "C:\\ProgramData\\Anaconda3\\envs\\tf-gpu\\lib\\site-packages\\sklearn\\neural_network\\_multilayer_perceptron.py:696: ConvergenceWarning: Stochastic Optimizer: Maximum iterations (100) reached and the optimization hasn't converged yet.\n", " ConvergenceWarning,\n" ] }, { "data": { "text/plain": [ "MLPClassifier(hidden_layer_sizes=(50, 20, 10), learning_rate_init=0.1,\n", " max_iter=100, random_state=1, solver='sgd', verbose=10)" ] }, "execution_count": 3, "metadata": {}, "output_type": "execute_result" } ], "source": [ "# Séparation les données en ensembles d'entraînement et de test (train/test)\n", "X_train, X_test = ...\n", "y_train, y_test = ...\n", "classifier = ...\n", "\n", "# Entraîner le modèle sur les données d'entraînement\n", "..." ] }, { "cell_type": "markdown", "id": "751221fd", "metadata": {}, "source": [ "# 5.\tÉvaluation du modèle" ] }, { "cell_type": "markdown", "id": "b24f7db5", "metadata": {}, "source": [ "\n", "### Question:\n", "Maintenant, évaluer le modèle. Vous allez estimer la précision moyenne des données et des étiquettes d'entraînement et de test." ] }, { "cell_type": "code", "execution_count": 4, "id": "d901fd77", "metadata": {}, "outputs": [ { "name": "stdout", "output_type": "stream", "text": [ "Score de l'ensemble d'entraînement: 0.997750\n", "Score de l'ensemble de test: 0.968300\n" ] } ], "source": [ "print(\"Score de l'ensemble d'entraînement: %f\" % ...)\n", "print(\"Score de l'ensemble de test: %f\" % ...)" ] }, { "cell_type": "markdown", "id": "050f14e9", "metadata": {}, "source": [ "# 6. Visualisation de l'évolution de la fonction de coût" ] }, { "cell_type": "markdown", "id": "e69cb11f", "metadata": {}, "source": [ "### Question:\n", "Afficher un graphe afin de visulaiser à quelle vitesse l'erreur a-t-elle diminué pendant l'entraînement." ] }, { "cell_type": "code", "execution_count": 5, "id": "c93abf9c", "metadata": {}, "outputs": [ { "data": { "image/png": "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\n", "text/plain": [ "
" ] }, "metadata": {}, "output_type": "display_data" } ], "source": [ "fig, axes = plt.subplots(..., ...)\n", "axes.plot(..., 'o-')\n", "axes.set_xlabel(...)\n", "axes.set_ylabel(...)\n", "plt.show()" ] }, { "cell_type": "markdown", "id": "19751aae", "metadata": {}, "source": [ "Ici, nous voyons que l’erreur diminue très rapidement pendant l'entraînement et elle se stabilise après la 40ème itération (rappelez-vous que nous avons défini 100 itérations au maximum en tant qu'hyperparamètre)." ] }, { "cell_type": "markdown", "id": "bca5e82a", "metadata": {}, "source": [ "# 7. Visualisation des poids estimés" ] }, { "cell_type": "markdown", "id": "08367d51", "metadata": {}, "source": [ "Ici, nous devons d'abord comprendre comment les poids (paramètres appris du modèle pour chaque couche) sont stockés." ] }, { "cell_type": "markdown", "id": "856149b2", "metadata": {}, "source": [ "Selon la documentation, l'attribut classifier.coefs_ est une liste de forme (n_layers-1,) de tableaux de poids, où la matrice de poids à l'indice i représente les poids entre la couche i et la couche i+1. Dans cet exemple, nous avons défini 3 couches cachées et nous avons également la couche d'entrée et de sortie. Nous nous attendons donc à avoir 4 tableaux de poids pour les poids entre les couches (entrée-L1, L1-L2, L2-L3 et L3-sortie)." ] }, { "cell_type": "markdown", "id": "e9d2edcc", "metadata": {}, "source": [ "De même, classifier.intercepts_ est une liste de vecteurs de biais, où le vecteur à l'indice i représente les valeurs de biais ajoutées à la couche i+1." ] }, { "cell_type": "markdown", "id": "30be7ed1", "metadata": {}, "source": [ "### Question:\n", "Ecrire la commande Python qui permet de vérifier cela." ] }, { "cell_type": "code", "execution_count": 6, "id": "773b5d89", "metadata": {}, "outputs": [ { "data": { "text/plain": [ "True" ] }, "execution_count": 6, "metadata": {}, "output_type": "execute_result" } ], "source": [ "..." ] }, { "cell_type": "markdown", "id": "7820ff3a", "metadata": {}, "source": [ "Rappel : la matrice de poids de la couche d'entrée aura une forme de 784 x #neurones_dans_la_1ère_couche_cachée. La matrice de poids de la couche de sortie aura une forme de #neurones_dans_la_3ème_couche_cachée x #nombre_de_classes.\n", "Visualisation des poids appris de la couche d'entrée.\n", "\n", "### Question:\n", "Ecrire les commandes Python qui permettent de visualiser les poids estimés." ] }, { "cell_type": "code", "execution_count": 7, "id": "bdf7bce7", "metadata": {}, "outputs": [ { "data": { "image/png": "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\n", "text/plain": [ "
" ] }, "metadata": {}, "output_type": "display_data" } ], "source": [ "target_layer = 0 #0 représente la couche d'entrée, 1 la 1ère couche cachée, etc.\n", "fig, axes = plt.subplots(1, 1, figsize=(15,6))\n", "axes.imshow(..., cmap=..., aspect=...)\n", "axes.set_xlabel(f\" nombre de neurones dans {target_layer}\")\n", "axes.set_ylabel(\"neurones dans la couche de sortie \")\n", "plt.show()" ] }, { "cell_type": "markdown", "id": "287dc5d1", "metadata": {}, "source": [ "\n", "### Question:\n", "Ecrire la commande Python qui permet de reformater les poids et les représenter en tant qu'images 2D à nouveau." ] }, { "cell_type": "code", "execution_count": 8, "id": "89d589de", "metadata": {}, "outputs": [ { "data": { "image/png": "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\n", "text/plain": [ "
" ] }, "metadata": {}, "output_type": "display_data" } ], "source": [ "# choisir la couche à représenter\n", "target_layer = 0 #0 représente la couche d'entrée, 1 la 1ère couche cachée, etc.\n", "fig, axes = plt.subplots(4, 4)\n", "vmin, vmax = classifier.coefs_[0].min(), classifier.coefs_[target_layer].max()\n", "for coef, ax in zip(classifier.coefs_[0].T, axes.ravel()):\n", " ax.matshow(..., cmap=..., vmin=0.5 * vmin, vmax=0.5 * vmax)\n", " ax.set_xticks(())\n", " ax.set_yticks(())\n", "plt.show()" ] }, { "cell_type": "markdown", "id": "a8463b2f", "metadata": {}, "source": [ "# 8. Conclusion" ] }, { "cell_type": "markdown", "id": "579b84c9", "metadata": {}, "source": [ "Le classificateur MLP est un modèle de réseau neuronal très puissant qui permet l'apprentissage de fonctions non linéaires pour des données complexes. La méthode utilise la propagation avant pour construire les poids et calcule ensuite la fonction d’erreur. Ensuite, la rétropropagation est utilisée pour mettre à jour les poids de manière à réduire l’erreur. Cela se fait de manière itérative et le nombre d'itérations est un hyperparamètre d'entrée. D'autres hyperparamètres importants sont le nombre de neurones dans chaque couche cachée et le nombre total de couches cachées. Ils doivent être affinés." ] } ], "metadata": { "kernelspec": { "display_name": "Python 3 (ipykernel)", "language": "python", "name": "python3" }, "language_info": { "codemirror_mode": { "name": "ipython", "version": 3 }, "file_extension": ".py", "mimetype": "text/x-python", "name": "python", "nbconvert_exporter": "python", "pygments_lexer": "ipython3", "version": "3.7.13" } }, "nbformat": 4, "nbformat_minor": 5 }